what amount is 10% more than RS. 90?​

Answers

Answer 1

Answer:

99

Step-by-step explanation:

x = 90 x 10/100 + 90

=> x = 900/100 + 90

=> x = 9 + 90

=> x = 99


Related Questions

please help!!!! Use the graph to complete the statement. O is the origin. Ry−axis ο Ry=x: (-1,2)
A. (1, -2) B. (-1, -2) C. (2, -1) D. (-2, -1)

Answers

Answer:  D. (-2, -1)

Step-by-step explanation:

Here we do two reflections to the point (-1, 2).

First, we do a reflection over the line x = y.  Remember that a reflection over a line keeps constant the distance between our point and the given line, so we have that for a pint (x, y), the reflection over the line y = x is:

Ry=x (x, y) = (y, x)

so for our point, we have:

Ry=x (-1, 2) = (2, -1)

Now we do a reflection over the y-axis, again, a reflection over a line keeps constant the distance between our point and the given line, so if we have a point (x,y) and we do a reflection over the y-axis, our new point will be:

Ry-axis (x,y) = (-x, y)

Then in our case:

Ry-axis (2, -1) = (-2, -1)

The correct option is D.

When a boy pulls his sled with a rope, the rope makes an angle of 70° with the horizontal. If a pull of 20 pounds on the rope is needed to move the sled, what is the horizontal component force?

7 lb
19 lb
20 lb
24 lb

Answers

Answer:

The answer to your question is 7 lbs.

Step-by-step explanation:

Data

angle = 70°

force = 20 lb

horizontal component

Process

To solve this problem just imagine a right triangle, where the force is the hypotenuse and the horizontal component is the adjacent side.

1.- Look for a trigonometric function that relates the adjacent side and the hypotenuse.

                  cos Ф = adjacent side / hypotenuse

- Solve for the adjacent side

                  adjacent side = hypotenuse x cosФ

- Substitution

                  adjacent side = 20 x cos 70

- Simplification

                  adjacent side = 20 x 0.342

- Result

                  adjacent side = 6.84 ≈ 7 lb  

a submarine is 50 meters below sea level. It goes up 15 meters and down 40 meters. What is the submarines new position relative to sea level

Answers

Answer:

-75 m

Step-by-step explanation:

Use sea level as the reference point.  

Initially the position is -50 m.

If the submarine goes up 15 m, then its new position is (-50 + 15) m, or -35 m.

If the sub now descends 40 m, its new position is (-35 - 40) m, or -75 m

rational or irrational? and why

Answers

Answer:

rational, simplified number is -0.5

Step-by-step explanation:

(see screenshot)

Can i please get some help im pretty desperate and will give brainiest to first answer with some steps, just try your best please

Answers

Answer:

To calculate the vertex you can use the vertex form ( i searched about the vertex and found it) but i don't know how to use it so i used another method.

The vertex is where the function inverts (y values) so it is going to be or a maximum or a minimum of the function. To calculate the min/max you derivate the function and then equal it to zero.

1) y  = -3x^2 -12x -10

the derivative of y is y' = -6x -12 = -6(x+2)

y'=0 <=>  -6(x+2) = 0 <=> x+2=0 <=> x=-2

y(-2) = -3(-2)^2 -12*(-2) -10 = 2

So the vertex is the point (-2,2) .

Because the slope of the equation is negative that means that the vertex is going to be the maximum , so the maximum is (-2,2) .

To find another 4 points you just have to pick values of x and replace them in the equation y  = -3x^2 -12x -10  to find y.

2) y = -2x^2 -12x -16

Do the same thing we did in point 1

y'= -4x-12 = -4(x+3)

y'=0 <=>  -4(x+3) = 0 <=> x+3=0 <=> x=-3

y(-3) = -2(-3)^2 -12*(-3) -16 = 2

So the vertex is the point (-3,2) .

Bob has a pyramid with a volume of 14. He'd like to create a second pyramid with a volume of 28. Which would guarantee he gets the desired
result?
O Double each side of the base of the pyramid.
O Double one side of the base of the pyramid.
O Double the slant height.
• Double the height.

Answers

Answer:

D

Step-by-step explanation:

Doubling the height will guarantee that the volume gets twice since V=(1/3)*B*h

If Bob wants to create a pyramid of volume 28 then he needs to double the height of the pyramid.

What is a pyramid?

Design or construction with a base that is a polygon and sides that are three or more triangles that come together at the top.

A pyramid is a 3D shape where 3 triangle comes and meet in such a way that the base side creates a triangle.

The volume of a pyramid is given by

Volume = (1/3)base²×height.

Let say

Base width is = w

Height is = h

Volume V = w²h/3

Given

14 = w²h/3

For volume to be 28

28 = w²(2h)/3

So it is clear that he needs to double the height.

To learn more about pyramids,

https://brainly.com/question/17615619

#SPJ2

prove that (3-4sin^2)(1-3tan^2)=(3-tan^2)(4cos^2-3)

Answers

Answer:

Proof in the explanation.

Step-by-step explanation:

I expanded both sides as a first step. (You may use foil here if you wish and if you use that term.)

This means we want to show the following:

[tex]3-9\tan^2(\theta)-4\sin^2(\theta)+12\sin^2(\theta)\tan^2(\theta)[/tex]

[tex]=12\cos^2(\theta)-9-4\cos^2(\theta)\tan^2(\theta)+3\tan^2(\theta)[/tex].

After this I played with only the left hand side to get it to match the right hand side.

One of the first things I notice we had sine squared's on left side and no sine squared's on the other. I wanted this out. I see there were cosine squared's on the right. Thus, I began with Pythagorean Theorem here.

[tex]3-9\tan^2(\theta)-4\sin^2(\theta)+12\sin^2(\theta)\tan^2(\theta)[/tex]

[tex]3-9\tan^2(\theta)-4(1-\cos^2(\theta))+12\sin^2(\theta)\tan^2(\theta)[tex]

Distribute:

[tex]3-9\tan^2(\theta)-4+4\cos^2(\theta)+12\sin^2(\theta)\tan^2(\theta)[/tex]

Combine like terms and reorder left side to organize it based on right side:

[tex]4\cos^2(\theta)-1+12\sin^2(\theta)\tan^2(\theta)-9\tan^2(\theta)[/tex]

After doing this, I since that on the left we had products of sine squared and tangent squared but on the right we had products of cosine squared and tangent squared. This problem could easily be fixed with Pythagorean Theorem again.

[tex]4\cos^2(\theta)-1+12\sin^2(\theta)\tan^2(\theta)-9\tan^2(\theta)[/tex]

[tex]4\cos^2(\theta)-1+12(1-\cos^2(\theta))\tan^2(\theta)-9\tan^2(\theta)[/tex]

Distribute:

[tex]4\cos^2(\theta)-1+12\tan^2(\theta)-12\cos^2(\theta)\tan^2(\theta)-9\tan^2(\theta)[/tex]

Combined like terms while keeping the same organization as the right:

[tex]4\cos^2(\theta)-1+3\tan^2(\theta)-12\cos^2(\theta)\tan^2(\theta)-9\tan^2(\theta)[/tex]

We do not have the same amount of the mentioned products in the previous step on both sides. So I rewrote this term as a sum. I did this as follows:

[tex]4\cos^2(\theta)-1+3\tan^2(\theta)-12\cos^2(\theta)\tan^2(\theta)-9\tan^2(\theta)[/tex]

[tex]4\cos^2(\theta)-1+3\tan^2(\theta)-4\cos^2(\theta)\tan^2(\theta)-8\cos^2(\theta)\tan^2(\theta)-9\tan^2(\theta)[/tex]

Here, I decide to use the following identity [tex]\cos\theta)\tan(\theta)=\sin(\theta)[/tex]. The reason for this is because I certainly didn't need those extra products of cosine squared and tangent squared as I didn't have them on the right side.

[tex]4\cos^2(\theta)-1+3\tan^2(\theta)-4\cos^2(\theta)\tan^2(\theta)-8\cos^2(\theta)\tan^2(\theta)-9\tan^2(\theta)[/tex]

[tex]4\cos^2(\theta)-1+3\tan^2(\theta)-4\cos^2(\theta)\tan^2(\theta)-8\sin^2(\theta)-9\tan^2(\theta)[/tex]

We are again back at having sine squared's on this side and only cosine squared's on the other. We will use Pythagorean Theorem again here.

[tex]4\cos^2(\theta)-1+3\tan^2(\theta)-4\cos^2(\theta)\tan^2(\theta)-8\sin^2(\theta)-9\tan^2(\theta)[/tex]

[tex]4\cos^2(\theta)-1+3\tan^2(\theta)-4\cos^2(\theta)\tan^2(\theta)-8(1-\cos^2(\theta))-9\tan^2(\theta)[/tex]

Distribute:

[tex]4\cos^2(\theta)-1+3\tan^2(\theta)-4\cos^2(\theta)\tan^2(\theta)-8+8\cos^2(\theta)-9\tan^2(\theta)[/tex]

Combine like terms:

[tex]12\cos^2(\theta)-9+3tan^2(\theta)-4\cos^2(\theta)\tan^2(\theta)[/tex]

Reorder again to fit right side:

[tex]12\cos^2(\theta)-9+4\cos^2(\theta)\tan(\theta)+3\tan^2(\theta)[/tex]

This does match the other side.

The proof is done.

Note: Reordering was done by commutative property.

Jack bought a car for $50,000. He spent $5000 on repairs. He sold the car at a profit of $5000. At what price did he sell the car.​

Answers

Answer:$60,000

Step-by-step explanation:

If he bought it for $50,000 and then spent $5,000 on repairs then he spent a total of $55,000. For a profit of $5,000 he would need to sell it for $60,000 Because

60,000 - 55,000 = 5,000

the area of a square poster is 27 in.2. Find the length of one side of the poster to the nearest tenth of an inc

Answers

Answer:

5.2 inches

Step-by-step explanation:

Let a be the side of the square poster.

The area of it is:

● A = a^2

● 27 = a^2

● a = √27

● a = √(9×3)

● a = 3√3

● a = 5.19

Round it to the nearest tenth

● a = 5.2 inches

The length of one side of the poster is approximately 5 inches rounded to nearest tenth.

What is rounding of numbers ?

We round numbers because they are easy to deal with and they also retain their value to that place they have been rounded.

According to the given question the area of a square poster is 27inch².

We have to determine the length of one side of the poster.

We know area of a square is (side)².

∴ (side)² = 27.

side = [tex]\sqrt{27}[/tex].

We know [tex]\sqrt{25}[/tex] is 5 and [tex]\sqrt{36}[/tex] is 6 so [tex]\sqrt{27}[/tex] will lie between 5 and 8 and much closer to 5.

learn more about rounding of numbers here :

https://brainly.com/question/15235224

#SPJ2

in this figure ST || QR, PT = 4cm, TR = 8cm, area of PQR = 90cm^2. Find PT:TR please help me ​

Answers

Answer: The required ratio is PT:TR  = 1:2.

Step-by-step explanation:

Given: In triangle PQR, ST || QR, PT = 4cm, TR = 8cm, area of PQR = 90 cm².

To find : PT:TR

.i.e. Ratio of PT to TR.

Here, PT:TR[tex]=\dfrac{PT}{TR}[/tex]

[tex]=\dfrac{4\ cm}{8\ cm}[/tex]

Divide numerator and denominator by 4 , we get

[tex]\dfrac{1}{2}[/tex]

Therefore, the required ratio is PT:TR  = 1:2.

PLEASE HELP ME!!!!!!!!!
The relationship between hours exercised and the score on a memory test is modeled by the following line of best fit: y
= 4.8x + 27. Interpret the slope and intercept of the trend line within the context of the data.

Answers

X= -5.625 I think that what it is at least. Hope it helps :(

Answer:

Slope: 4.8, x-intercept: -27/4.8, y-intercept: 27

Step-by-step explanation:

To find the slope, use the formula y = mx + b. In this formula, m represents the slope. Since the given equation is based on the above formula, m is 4.8.

To find the x-intercept, set y equal to 0 and solve for x.

y = 4.8x + 27

0 = 4.8x + 27

-27 = 4.8x

-27/4.8 = x

x = -27/4.8

To find the y-intercept, we use the formula y = mx + b again. In this formula, b represents the y-intercept. And, since the equation is based on this formula, b is 27.

(Remember this for future reference: even though the formula says +b, the y-intercept can still be negative).

I hope this helped!

what is the graph of the following function??
PLEASE HELP!!

Answers

Answer:

umm it's a piecewise function and I think the graph is correct I don't really understand what you mean by what is the "graph" of the following function since the graph is already there...

Step-by-step explanation:

 Which table below matches the scatter plot?



x 3 4 6 1 7 8 1 2
y 5 9 3 2 7 6 4 5

x 2 1 8 6 7 1 4 3
y 5 9 3 2 7 6 4 5

x 3 4 6 1 7 8 1 2
y 1 2 3 4 5 6 7 8

x 3 4 6 1 7 8 1 2
y 2 1 7 1 3 5 6 4

Answers

Answer:

The first table

x 3 4 6 1 7 8 1 2

y 5 9 3 2 7 6 4 5

How do u find the area of a rectangle?

Answers

Answer:

6x-21

Step-by-step explanation:

Area of a rectangle is length times width.

3*(2x-7)-

6x-21

Answer:

6x - 21

Step-by-step explanation:

Since area = L x W (Length x Width)

--> 3(2x - 7)

6x - 21

if three less than one half a number is equal to one-third of the same number, find the number​

Answers

Answer: The number would be 18

Step-by-step explanation:

18/2=9 9-3=6 6*3=18

The equation be (x/2) - 3 = x/3 then, the number of x = 18.

How to estimate the value of x?

To estimate the value of x, bring the variable to the left side and bring all the remaining values to the right side. Simplify the values to estimate the result.

Let x be the number

From the given information, we get

(x/2) - 3 = x/3

The set all the fractions on one side and constants on another side

(x/2) - (x/3) = 3

Multiply by LCM

3x - 18 = 2x

Add 18 to both sides

3x - 18 + 18 = 2x + 18

simplify

3x = 2x + 18

Subtract 2x from both sides

3x - 2x = 2x + 18 - 2x

x =18.

Therefore, the number is x = 18.

To learn more about algebraic expression refer to:

https://brainly.com/question/4541471

#SPJ2

a number has 2,5 and 7 as its prime factors. what are the four smallest values it and take​

Answers

Answer:

  70, 140, 280, 350

Step-by-step explanation:

Obviously, it must have the factors 2, 5, 7 as a minimum, so the smallest value is 2×5×7 = 70.

Any of these primes can be added to the product. In increasing order, the smallest additional factors will be 2, 4, 5, 7, 8, 10, ...

So, the four smallest numbers with prime factors of 2, 5, and 7 are ...

  70 = 2·5·7

  140 = 2²·5·7

  280 = 2³·5·7

  350 = 2·5²·7

Anyone want to help...?

Answers

Answer:

-1

Step-by-step explanation:

3/2 * (-22/33)

Simplify by dividing the second fraction by 11

3/2 * (-2/3)

Rewriting

3/3 * (-2/2)

-1/1

Answer:

-1

Step-by-step explanation:

(a/b)(c/d) = (a*c)(

(3/2)(-22/33)

(3*-22)/(2*33) = -66/66 = -1

Type the correct answer in each box
the value of x is ____ and the value of y is ____ . HELP ME PLEASE

Answers

Here,

x+50°+70°= 180° [°.°Angle sum of triangle]

↪x= 180°-120°

↪x=60°

.°. x = 60°

Now,

Let z be the unknown angle of the second triangle

z+60°+50°=180° [°.° Angle sum of triangle ]

↪z=180°-110°

↪z=70°

. ° . z=70°

Then,

z+x+y= 180°

↪70°+60°+y=180°

↪y=180°-130°

↪y=50°

.°.y = 50°

Answer:

x = 60°, y = 50°

Step-by-step explanation

We know: the angles measures in a triangle add up to 180°.

(look at the picture)

z + 50 + 60 = 180

z + 110 = 180    subtract 110 from both sides

z = 70°

x + 50 + 70 = 180

x + 120 = 180         subtract 120 from both sides

x = 60°

Angles x, y, and z are on one side of a straight line. Therefore they add to 180°.

x + y + z = 180

60 + y + 70 = 180

y + 130 = 180         subtract 130 from both sides

y = 50°

Which two ratios represent quantities that are proportional? A 25/28 and 5/7 B 22/33 and 14/21 C 16/13 and 13/16 D 48/60 and 35/42​

Answers

Answer:

Step-by-step explanation:

the third term and the fifth term of a geometric progression are 2 and 1/8 respectively. If all terms are positive, find the sum to the infinity of the progression​

Answers

Answer:

42 + 2/3

Step-by-step explanation:

First, to calculate the sum of an infinite geometric series, our formula is

a₁/(1-r), with a₁ being the first term of the series and r being the common ratio. Therefore, we want to find both a₁ and r.

To find r, we can first determine that 2 * r = a₄ and a₄ * r = a₅, as the ratio separates one number from the next in a geometric series. Therefore, we have

2 * r * r = a₅

2 * r² = 1/8

divide both sides by 2 to isolate the r²

r² = 1/16

square root both sides to isolate r

r =± 1/4. Note the ± because r²=1/16 regardless of whether r = 1/4 or -1/4. However, because all terms are positive, r must be positive as well, or a₄, for example, would be 2 * (-1/4) = -0.5

Therefore, r = 1/4 .

To find the first term, we know that a₁ * r = a₂, and a₂ * r = a₃. Therefore, a₁ * r² = a₃ = 2

a₁ * 1/16 = 2

divide both sides by 1/16 to isolate a₁

a₁ = 2 * 1/ (1/16)

= 2 * 16

= 32

Plugging a₁ and r into our infinite geometric series formula, we have

a₁/(1-r)

= 32  / (1-1/4)

= 32/ (3/4)

= 32/ 0.75

= 42 + 2/3

Shawn wanted to model the number 13,450 using 13,450 using base-ten blocks how many large cubes, flats, and longs does he need to model the number

Answers

Answer:

See attached

Step-by-step explanation:

13450 = 13000 + 400 + 50

13*1000  = 13 large cubes4*100      = 4 flats5*10        = 5 longs

The graph of y=3x^2-3x-1 is shown. Use the graph to find estimates for the solutions of i)3x^2-3x+2=2 ii) 3x^2-3x-1=x+1

Answers

Intersection point of graph of function is known as solution of the function.

Graph is attached below, in which solution is shown.

1. Here, given that  [tex]3x^2-3x+2=2[/tex]

It can be written as,   [tex]y=3x^2-3x+2\\\\y=2[/tex]

Intersection point of graph of above two equation will be the solution of given function,

Solutions are (1, 2) and (0, 2)

2. Given that , [tex]3x^2-3x-1=x+1[/tex]

It can be written as

                [tex]y=3x^2-3x-1\\\\y=x+1[/tex]

Intersection of graph of above two equation will be the solution of given equation.

Solutions are  (1.721, 2.721) and (- 0.387, 0.613)

Both graph attached below,

Learn more:

https://brainly.com/question/13713063

20. A pool holds 1440 CUBIC feet of water, the aty
charges $). 15 per cubic meter of water
used now, much will it cost to fill the pool?

Answers

Complete question :

A pool holds 1440 cubic feet of water, the city charges $1.75 per cubic meter of water used.

How much will it cost to fill the pool?

Answer:

$71.36

Step-by-step explanation:

Amount charged per cubic meter = $1. 75

Total volume of pool = 1440 ft^3

Using the conversion unit chart :

1 cubic meter = 35.3147 cubic foot

Then we can express the capacity of water held by the pool in cubic metre.

If 1 cubic meter = 35.3147 cubic foot

Then ;

y cubic meter = 1440 cubic foot

y = 1440/35.3147

y = 40.776220 cubic meter

Thus;

Cost of filling the pool will be;

$1.75 × 40.776220

= $71.358386

= $71.36

pls i want help in this sum​

Answers

Step-by-step explanation: The first rectangle to the right is 24 because 10+14 equals 24. The rectangle to the left is 19 because 10+9 equals 19. Finally the rectangle on the bottom is 23 because 14+9 is 23

An Aquarium’s dimensions are 3 1/4 ft x 2 ft x 1 3/4. What is the volume of the Aquarium?

Answers

Volume = length x width x height

Volume = 3 1/4 x 2 x 1 3/4 = 11 3/8 cubic feet

Answer:

11 3/8 ft^3

Step-by-step explanation:

The volume is given by

V = l*w*h

V = 3 1/4 * 2 * 1 3/4

Change the number to improper fractions

V = (4*3+1)/4  *2 * (4*1+3)/4

   = 13/4 *2* 7/4

Rewriting

   = 13/1 * 2/4 * 7/4

    = 13/1 * 1/2 *7/4

    = 91/8

Changing back to a mixed number

8 goes into 91  11 times with 3 left over

11 3/8 ft^3

What is the constant of proportionality in the equation: y=7x ...?

Answers

To get y, we have to multiply x by 2. In your question, the constant of proportionality already jump out at us which is to get y, we have to multiply x by 7. Therefore the constant of proportionality is y=7x.


Hello!! Please help me solve the question, could you please also tell me which calculator we can use? Thank you

Answers

You can use any calculator in Maths.

One angle of a triangle has a measure of 66 degrees. The measure of the third angle is 57 degrees more than 1/2 the measure of the second angle. The sum of the angle measures of a triangle is 180 degrees. What is the Measure of the second angle? What is the measure of the third angle?

Answers

Answer:

76 degrees

Step-by-step explanation:

Let the angles be x, y and z

As per given:

x= 66z = 57 + 1/2yx+y+z = 180

Considering the first 2 equations in the third one:

66 + y + 57 + 1/2y = 1801.5y = 180 - (66+57)1.5y = 57y = 57/1.5y = 38

So the third angle is:

z = 57 + 38/2 = 76 degrees

How much water will be in each bottle if the total amount of water is equally divided among the bottles?

Answers

Answer:

D

Step-by-step explanation:

add everything up devide by 3

Answer:

C. 1/2 liter

Step-by-step explanation:

First, find the total amount of water by multiplying the amounts of water by the number of bottles, then adding all 3 values together:

1/4(1) = 1/4 liter

1/2(4) = 2 liters

3/4(1) = 3/4 liter

1/4 + 2 + 3/4 = 3 liters

There are 6 bottles in total, if we count the number of dots.

To find how much water would be in each bottle if the water was equally divided, divide the total amount of water by the number of bottles:

3 / 6 = 1/2

= 1/2 liter

Which of the following statements is true about odd
and/or even numbers?
F. The sum of any 2 even numbers is odd.
G. The sum of any 2 odd numbers is ood.
H. The quotient of any 2 even numbers is odd.
J. The quotient of any 2 even numbers is even.
K. The product of any 2 odd numbers is odd.

Answers

Answer:

k) the product of ant 2 odd numbers is odd

I don't know.

Let's check um out.

For each choice, I'll try to find an example to show that it's false.

F. The sum of any 2 even numbers is odd.  

    2+2=4. 4 is even. This one is false.

G. The sum of any 2 odd numbers is ood.

    3+3=6.  6 is even.  This one is false.

H. The quotient of any 2 even numbers is odd.

    8÷4=2. 2 is even. 10÷4=2.5.  2.5 is neither odd nor even. This one is false.

J. The quotient of any 2 even numbers is even.

   10÷2=5.  5 is odd.  This one is false.

K. The product of any 2 odd numbers is odd.

    I can't find an example where this is false.

    So I'm gonna say that this is the true one.

Other Questions
Christine's gross monthly salary at her jobis $5,250. She has the followingdeductions from her paycheck.What is Christine's net take-home pay permonth? HELP MEEEE FAST PLZMY MOM THINKS IM SMART BUT I DON'T GET THIS AHHHH HELP introduction of aldolf Hitler What do you think Native Americans believed about who owned this land and how it should be used? I need help with this guys ! I need help ASAP!! Please explain how to solve the problem Escoge la mejor traduccin para la siguiente oracin. They have written a long letter. A. Han escribo una carta larga B. Han escribito una carta larga. C. Han escribido una carta larga. D. Han escrito una carta larga. 2. According to this article, whathappens to a person's sense ofindividuality when they follow alarger group?A It shuts down.B It grows weaker.C It grows stronger.D It remains the same. im pretty bad at math Problem 25.40 What is the energy (in eV) of a photon of visible light that has a wavelength of 500 nm Which equation represents the data shown in the table provided in the image? A. y = 2x + 1B. y = 3x 1C. y = 2.5xD. y = 2.5x + 1Please include ALL work! IPv6 can use a DHCPv6 server for the allocation of IPv6 addressing to hosts. Another IPv6 addressing option utilizes the IPv6 Neighbor Discovery Protocol (NDP) to discover the first portion of the IPv6 address (network prefix) from local routers, and the host can create its own host ID. What standard is commonly used by an IPv6 host to generate its own 64-bit host ID in journey of sparrows when was it man versus man Divers often inflate heavy duty balloons attached to salvage items on the sea floor. If a balloon is filled to a volume of 1.20 L at a pressure of 6.25 atm, what is the volume of the balloon when it reaches the surface? Which statement most accurately describes what the medicine bag symbolizes to Martins grandpa? It symbolizes his connection to his family members and his way of life. It symbolizes his hope that Martin will one day live on the reservation. It symbolizes the sacred dream he had during his vision quest. It symbolizes the deer that his father had killed long ago. the moon revolves around the earth in a nearly circular orbit kept by gravitational force exerted by the earth work done will be Calgary Industries is preparing a budgeted income statement for 2018 and has accumulated the following information. Predicted sales for the year are $695,000 and cost of goods sold is 40% of sales. The expected selling expenses are $77,500 and the expected general and administrative expenses are $86,500, which includes $19,500 of depreciation. The company's income tax rate is 30%. The budgeted net income for 2018 is: Which statement correctly describes the quantitative relationship between acceleration and net force on an object shown in the data table? (1 point) When the net force applied to an object changes, the mass of the object remains constant. When the net force applied to an object changes, the acceleration changes by a different factor. When the net force applied to an object changes, the mass of the object also changes. When the net force applied to an object changes, the acceleration changes by the same factor. Estimate the mean exam score for the 50 students in Prof. Burke's class.Scoref40 but less than 502150 but less than 603960 but less than 704070 but less than 803480 but less than 9028Total162Group of answer choices63.7862.3464.8965.56 The mass of earth 6*10^24kg and radius is 400kg Now find the value of acceleration due to gravity when a object is 3600km from the earth surface PLZZZ help!!!Find the area and the perimeter of the shaded regions below. Give your answer as a completely simplified exact value in terms of (no approximations). The figures below are based on semicircles or quarter circles and problems b), c), and d) are involving portions of a square.